Đến nội dung

Ankh nội dung

Có 84 mục bởi Ankh (Tìm giới hạn từ 05-06-2020)



Sắp theo                Sắp xếp  

#626256 Inequalities From 2016 Mathematical Olympiads

Đã gửi bởi Ankh on 10-04-2016 - 09:07 trong Bất đẳng thức - Cực trị

Bài 18 (Macedonia National Olympiad). Cho $n \;(n \geqslant 3)$ số thực dương $a_1,\,a_2,\,\ldots,a_n$ thỏa mãn điều kiện

$$\frac{1}{1+a_1^4} + \frac{1}{1+a_2^4} + \cdots + \frac{1}{1+a_n^4} = 1.$$

Chứng minh rằng $$a_1a_2 \cdots a_n \ge (n-1)^{\frac n4}.$$
 

 Bài này quen thuộc

 Áp dụng bất đẳng thức AM-GM ta có $\dfrac{a_1^4}{1+a_1^4}=\dfrac{1}{1+a_2^4}+\dfrac{1}{1+a_3^4}+...+\dfrac{1}{a_n^4}\geq (n-1)\sqrt[n-1]{\dfrac{1}{(1+a_2^4)(1+a_3^4)...(1+a_n^4)}}$

 Tương tự thì $\begin{matrix}\dfrac{a_2^4}{1+a_2^4}\geq (n-1)\sqrt[n-1]{\dfrac{1}{(1+a_1^4)(1+a_3^4)...(1+a_n^4)}}\\ \dfrac{a_3^4}{1+a_3^4}\geq (n-1)\sqrt[n-1]{\dfrac{1}{(1+a_1^4)(1+a_2^4)...(1+a_n^4)}} \\... \\ \dfrac{a_n^4}{1+a_n^4}\geq (n-1)\sqrt[n-1]{\dfrac{1}{(1+a_1^4)(1+a_2^4)...(1+a_{n-1}^4)}}\end{matrix}$

 Nhân các bất đẳng thức trên lại ta được $\prod _{i=1}^n \dfrac{a_i^4}{1+a_i^4}\geq (n-1)^n.\dfrac{1}{\prod \limits _{i=1}^n (1+a_i^4)}$

 Thu gọn và lấy căn 2 vế ta có điều cần chứng minh




#634795 Inequalities From 2016 Mathematical Olympiads

Đã gửi bởi Ankh on 22-05-2016 - 21:08 trong Bất đẳng thức - Cực trị

Bài 34 (Serbia Junior TST). Cho $a,b,c$ là ba số thực dương. Chứng minh rằng $$\frac{2a}{\sqrt{3a+b}}+\frac{2b}{\sqrt{3b+c}}+\frac{2c}{\sqrt{3c+a}}\leq \sqrt{3(a+b+c)}.$$
 

 Áp dụng bất đẳng thức Cauchy-Schwarz ta có $\sum \dfrac{2a}{\sqrt{3a+b}}\leq 2\sqrt{(a+b+c)\sum \dfrac{a}{3a+b}}$

 Nên ta chỉ cần chứng minh $\sum \dfrac{a}{3a+b}\leq \dfrac{3}{4}\Leftrightarrow \sum \dfrac{b}{3a+b}\geq \dfrac{3}{4}$

 Áp dụng bất đẳng thức Cauchy-Schwarz ta có $\sum \dfrac{b}{3a+b}=\sum \dfrac{b^2}{3ab+b^2}\geq \dfrac{(a+b+c)^2}{(a+b+c)^2+ab+bc+ca}\geq \dfrac{3}{4}$

 Dấu "=" xảy ra khi $a=b=c$




#638421 Inequalities From 2016 Mathematical Olympiads

Đã gửi bởi Ankh on 05-06-2016 - 23:21 trong Bất đẳng thức - Cực trị

Bài 39 (Mediterranean MO). Cho ba số thực dương $a,b,c$ thỏa mãn $a+b+c=3.$ Chứng minh rằng
\[\sqrt{\frac{b}{a^2+3}}+\sqrt{\frac{c}{b^2+3}}+\sqrt{\frac{a}{c^2+3}} \leqslant \frac32\sqrt[4]{\frac{1}{abc}}.\]

 

 Vế phải của bất đẳng thức thực ra là $\dfrac{1}{2}.\dfrac{3}{\sqrt[4]{abc}}=\dfrac{a+b+c}{2\sqrt[4]{abc}}$

 Áp dụng bất đẳng thức AM-GM thì $a^2+3\geq 4\sqrt{a}$ nên $VT\leq \sum \dfrac{\sqrt{b}}{2\sqrt[4]{a}}$

 Đặt $\sqrt[4]{a}=x,\cdots $ thì ta chỉ cần chứng minh $\sum \dfrac{y^2}{x}\leq \dfrac{x^4+y^4+z^4}{xyz}\Leftrightarrow x^3y+y^3z+z^3x\leq x^4+y^4+z^4$

 Bất đẳng thức trên chứng minh khá dễ bằng AM-GM :)




#624968 Inequalities From 2016 Mathematical Olympiads

Đã gửi bởi Ankh on 05-04-2016 - 00:29 trong Bất đẳng thức - Cực trị

 Bài 2. Ta chú ý đến một phân tích quen thuộc là $(x+y)^2=(x-y)^2+4xy$

 Bây giờ đặt $\sum _{j=1}^na_jb_j=S$ và $a_{i}b_{i}=S_i$ thì ta cần chứng minh $\sum_{i=1}^n \dfrac{(a_{i+1}+b_{i+1})^2}{n(a_i-b_i)^2+4(n-1)S} \geq \dfrac{1}{n-1}$

 Hay $\sum_{i=1}^n \dfrac{(a_{i+1}-b_{i+1})^2+4S_{i+1}}{n(a_i-b_i)^2+4(n-1)S} \geq \dfrac{1}{n-1}$

 Không mất tính tổng quát giả sử $(a_1-b_1)^2\leq \min \left \{(a_2-b_2)^2,(a_3-b_3)^2,...,(a_n-b_n)^2\right \}$ thì ta có

 $\sum_{i=1}^n \dfrac{(a_{i+1}-b_{i+1})^2+4S_{i+1}}{n(a_i-b_i)^2+4(n-1)S} \geq \dfrac{\sum \limits_{i=1}^n (a_{i}-b_{i})^2+4\sum \limits_{k=1}^nS_{k}}{n(a_1-b_1)^2+4(n-1)S}=\dfrac{\sum \limits_{i=1}^n (a_{i}-b_{i})^2+4S}{n(a_1-b_1)^2+4(n-1)S}$

 Nên ta chỉ cần chứng minh $\dfrac{\sum \limits_{i=1}^n (a_{i}-b_{i})^2+4S}{n(a_1-b_1)^2+4(n-1)S}\geq \dfrac{1}{n-1}\Leftrightarrow \sum_{i=1}^n (a_{i}-b_{i})^2\geq \dfrac{n}{n-1}(a_1-b_1)^2\Leftrightarrow \sum_{i=2}^n (a_{i}-b_{i})^2\geq \dfrac{1}{n-1}\geq (a_1-b_1)^2$

 Áp dụng bất đẳng thức Cauchy-Schwarz ta có $\sum_{i=2}^n (a_{i}-b_{i})^2\geq \dfrac{1}{n-1}\left (\sum_{j=2}^n a_j-\sum_{k=2}^n b_k \right )^2$

 Mặt khác $\sum_{i=1}^n (a_i-b_i)=0$ nên $\left (\sum_{j=2}^n a_j-\sum_{k=2}^n b_k \right )^2=(a_1-b_1)^2$

 Từ đó ta có điều cần chứng minh ~.~ Mỏi tay




#624952 Inequalities From 2016 Mathematical Olympiads

Đã gửi bởi Ankh on 04-04-2016 - 22:53 trong Bất đẳng thức - Cực trị

 Bài 11. Không giảm tính tổng quát giả sử $a>b>c$ ( hiển nhiên $a,\ b,\ c$ đôi một không đồng thời bằng nhau )

 Khi đó ta có $1=(a-b)(b-c)(a-c)\leq \dfrac{1}{4}(a-c)^3\Rightarrow a-c\geq \sqrt[3]{4}$

 Lại có $|a|+|b|+|c|\geq |a-c|+|b|\geq a-c\geq \sqrt[3]{4}$ nên ta có giá trị nhỏ nhất

 Dấu "=" xảy ra khi và chỉ khi $b=0;a=-c=\dfrac{1}{\sqrt[3]{2}}$

 Mà mình thấy có mấy câu hơi dở đi nhỉ  :wacko:




#646173 Inequalities From 2016 Mathematical Olympiads

Đã gửi bởi Ankh on 23-07-2016 - 21:52 trong Bất đẳng thức - Cực trị

Bài 52 (Taiwan TST). Tìm hằng số $k$ nhỏ nhất để bất đẳng thức

\[\frac{x^2y^2}{1-z}+\frac{y^2z^2}{1-x}+\frac{z^2x^2}{1-y}\leq k-3xyz,\]

luôn đúng với mọi số thực dương $x,y,z$ thỏa mãn điều kiện $x+y+z=1.$

 Áp dụng AM-GM ta có $\sum \dfrac{x^2y^2}{1-z}=\sum \dfrac{x^2y^2}{x+y}\leq \sum \dfrac{xy(x+y)}{4}$

 Nên ta sẽ có $\sum \dfrac{x^2y^2}{1-z}+3xyz\leq \dfrac{1}{4}\left(\sum xy(x+y)+12xyz\right)=\dfrac{1}{4}\left(xy+yz+zx+9xyz\right)\leq \dfrac{1}{6}$

 Hơn nữa tại $k=\dfrac{1}{6}$ thì bất đẳng thức xảy ra dấu "=" tại $x=y=z=1/3$ nên $k_{\min}=\dfrac{1}{6}$




#629340 Đề thi học sinh giỏi môn toán khối 10 khu vực DUYÊN HẢI VÀ ĐỒNG BẰNG BẮC BỘ n...

Đã gửi bởi Ankh on 24-04-2016 - 15:48 trong Thi HSG cấp Tỉnh, Thành phố. Olympic 30-4. Đề thi và kiểm tra đội tuyển các cấp.

 Câu 3 em nghĩ chỉ cần làm như sau

 Đặt $n=mq+r$ với $0<r<m$ thì khi xét trong $\text{mod 2016}$ ta có $2016^m\equiv -1\Rightarrow 2016^{mq+r}\equiv (-1)^q.2016^r\Rightarrow 2016^r.(-1)^q+1\vdots 2016^m+1$ 

 Mà ta có $|2016^r.(-1)^q+1|\leq |2016^r+1|\leq 2016^m+1$ nên điều trên xảy ra khi $q$ lẻ và $r=0$

 Vậy ta có điều cần chứng minh

 Câu bất đẳng thức thì dùng Cauchy Schwarz $\sum \dfrac{b}{a^2}.\sum \dfrac{1}{ab}.(1+1+1)\geq \left (\sum \dfrac{1}{a}\right )^3$ sau đó biến đổi tương đương cũng ra :)




#654369 Đề thi chọn đội tuyển quốc gia tỉnh Quảng Bình

Đã gửi bởi Ankh on 16-09-2016 - 11:02 trong Thi HSG cấp Tỉnh, Thành phố. Olympic 30-4. Đề thi và kiểm tra đội tuyển các cấp.

Bài hình ngày 1:

a, Bằng biến đổi góc ta có $DKG=FKE$ do đó $GKE=DKF=DPF$. Bằng phương tích suy ra $GDFE$ nội tiếp. Ta chứng minh được $(P,D,G)$ và $(P,F,E)$ thẳng hàng (bằng cộng góc). Từ các điều kiện trên ra có DPCM

Ai có lời giải câu b chưa vậy. Câu a khá đơn giản nhưng câu b làm tốn thời gian quá!! :(  :D

 Bạn ghi rõ đoạn chứng minh thằng hàng được không, mình cũng cộng góc mà không ra.

 P/s: Ai giải câu 5 đi




#639570 Đề tuyển sinh chuyên toán Quốc Học Huế 2016-2017

Đã gửi bởi Ankh on 11-06-2016 - 13:17 trong Tài liệu - Đề thi

Câu 4.

Untitled.png

 a) Ta có $\angle PKB=\angle 180^\circ -\angle MKB=180^\circ -\angle MAB=\angle BAN=\angle BLN=180^\circ -\angle BLP$ nên tứ giác $PKBL$ nội tiếp

 b) Ta có $\angle BKO_2=\angle BO_1O_2=\dfrac{1}{2}\angle BOA=\angle BKA$ nên $K,A,O_2$ thằng hàng, tương tự cho $L,A,O_1$

 Do đó mà $\angle KBA=\dfrac{1}{2}\angle KO_1A=\dfrac{180^\circ -\angle KAO_1}{2} =\dfrac{180^\circ -\angle LAO_2}{2}=\dfrac{1}{2}\angle LO_2A=\angle LBA$

 Nên $BA$ là phân giác $\angle KBL$, do đó $A$ cách đều $BK$ và $BL$

 c) Ta có $\angle PMN=\angle KBA=\angle LBA=\angle PNM$ nên tam giác $PMN$ cân tại $P$

 Nếu $P\in AB$ thì $PK.PM=PA.PB=PL.PN$ nên $PK=PL$

 Nếu $PK=PL$ thì $PK.PM=PL.PN$ nên nếu gọi $A'$ và $A''$ là giao $PB$ với $(O_1)$ và $(O_2)$ thì $PK.PM=PA'.PB=PL.PN=PA''.PB$

 Do đó $A'\equiv A''\equiv A$, và đó đó $P\in AB$




#639306 Đề thi tuyển sinh môn Toán (Chuyên) Quảng Bình năm 2016-2017

Đã gửi bởi Ankh on 10-06-2016 - 11:01 trong Tài liệu - Đề thi

Câu 4. 

a. Ta có $\angle NCM=\angle CAE=\angle NAM$ nên tứ giác đó nội tiếp

b. Có một kết quả quen thuộc là $\angle CAO+\angle ABC=90^\circ$, mà $\angle ABC=\angle AKD$ nên suy ra điều cần chứng minh

c. Sử dụng Thales do $EF\parallel CD$ nên $\dfrac{NF}{CN}=\dfrac{EF}{CD}\Rightarrow \dfrac{CF}{CN}=1-\dfrac{CF}{CD}$

 Chuyến vế ta có điều cần chứng minh

Hình gửi kèm

  • Untitled.png



#626090 ĐỀ THI OLYMPIC 24/3 TỈNH QUẢNG NAM

Đã gửi bởi Ankh on 09-04-2016 - 15:50 trong Thi HSG cấp Tỉnh, Thành phố. Olympic 30-4. Đề thi và kiểm tra đội tuyển các cấp.

Trích dẫn trungvmfcsp

 Cách của mình có lẽ gọn hơn xíu, như sau

 Áp dụng AM-GM ta có $3\sqrt[3]{z^2x^2}+1\leq xz+x+z+1=(x+1)(z+1)$ và $(2x+3y+z)^3=(x+2y+3)^3=(x+1+y+1+y+1)^3\geq 27(x+1)(y+1)^2$

 Áp dụng bất đẳng thức Cauchy-Schwarz ta có $P\geq 27\left [\dfrac{(y+1)^2}{z+1}+\dfrac{(z+1)^2}{x+1}+\dfrac{(x+1)^2}{y+1}\right ]\geq 27(x+y+z+3)=162$

 Dấu "=" xảy ra khi $x=y=z=1$




#670194 $f(x+y)+f(x)f(y)=f(xy)+f(x)+f(y)$

Đã gửi bởi Ankh on 28-01-2017 - 01:12 trong Phương trình hàm

Tìm $f:R\rightarrow R$ thỏa mãn:

$f(x+y)+f(x)f(y)=f(xy)+f(x)+f(y)$ (1)

Với mọi $x,y$ thực

 Thay $y:=0$ vào (1) ta được $f(0)(f(x)-2)=0$, cho tiếp $x:=0$ suy ra $f(0)=0$ hoặc $f(0)=2$

 Với $f(0)=2$ thì suy ra $f(x)\equiv 2$

 Với $f(0)=0$, thay $x,y$ trong (1) bởi 2 suy ra $f(2)(f(2)-2)=0$

  - Nếu $f(2)=0$, thay $y=x=1$ vào (1) suy ra $f(1)=0$ hoặc $f(1)=3$

    Với $f(1)=3$, thay $y$ lần lượt bởi $1$ và $2$ vào (1) suy ra $f(x+1)+f(x)=3$ và $f(x+2)=f(2x)+f(x)$, suy ra $f\equiv 0$, vô lí

    Với $f(1)=0$, thay $y:=1$ vào (1) suy ra $f(x+1)=2f(x)$, thay tiếp $x:=x+1$ vào (1) ta có

     $f(x+y+1)+f(x+1)f(y)=f(xy+y)+f(x+1)+f(y)\Leftrightarrow f(xy+y)=2f(xy)+f(y)$, hay $f(x+y)=2f(x)+f(y)=f(x)+2f(y)$, suy ra $f$ hằng hay $f\equiv 0$

  - Nếu $f(2)=2$, thay $x,y$ trong (1) bởi 1 suy ra $f(1)=1$ hoặc $f(1)=2$ 

    Với $f(1)=2$, thay $y:=1$ vào (1) suy ra $f\equiv 2$, vô lí 

    Với $f(1)=1$, thay $y:=1$ vào (1) suy ra $f(x+1)=f(x)+1$ 

    Lại thay $x:=x+1$ vào (1) và biến đổi suy ra $f(xy+y)=f(xy)+f(y)$ hay $f(x+y)=f(x)+f(y)$, suy ra $f(xy)=f(x)f(y)$, với mọi $x,y\in \mathbb{R}$

    Từ đây suy ra $f(x)=x$ 

 Vậy có 3 nghiệm hàm thỏa mãn $f(x)\equiv 0, f(x)\equiv 2, f(x)=x,\forall x\in \mathbb{R}$




#632833 Bài bất đẳng thức VMO 1995?

Đã gửi bởi Ankh on 13-05-2016 - 07:31 trong Bất đẳng thức - Cực trị

Bài này là TST chứ bạn  :mellow:




#637423 thảo luận THTT 03/2016

Đã gửi bởi Ankh on 01-06-2016 - 16:31 trong Toán học & Tuổi trẻ

Bài T9/465 giải như sau có vẻ gọn gàng hơn

Clipboardimage2016-06-01102549.png

P/s: Bài T12 mình giải không hay nên cũng không muốn post

 Một cách nữa cũng khá gọn :)

 Untitled.png

 Còn đây là lời giải của mình cho bài T10

 Untitled1.png




#640377 Bài Toán Dùng Sơ Đồ Cây

Đã gửi bởi Ankh on 14-06-2016 - 22:12 trong Các dạng toán THPT khác

Bài Toán:

 

Cho tập $ A= \{ 1; 2 ; 3; ...; 8 \}$ hỏi có thể có bao nhiêu số có 6 chữ số phân biệt được xếp theo trình tự từ bé đến lớn sao cho 6 chữ số này được chọn từ tập $A$ đã cho ?

 Em nghĩ chỉ việc chọn 6 số bất kì trong 8 số trên, còn việc sắp xếp thứ tự thì sau khi chọn do 6 số phân biệt nên mỗi cách chọn 6 số sẽ cho ra duy nhất một số và không lặp. 




#646360 $\frac{1}{5} \sum_{n=1}^{5} \sqrt[n]{\prod_{i=1}^{n}...

Đã gửi bởi Ankh on 24-07-2016 - 22:49 trong Bất đẳng thức và cực trị

Lời giải cho bài toán tổng quát :)

File gửi kèm  Kedlaya.pdf   224.76K   289 Số lần tải




#630221 Chứng minh rằng $\frac{1}{a^2}+\frac{...

Đã gửi bởi Ankh on 29-04-2016 - 20:28 trong Bất đẳng thức và cực trị

Cho các số thực dương a,b,c thỏa mãn abc=1

Chứng minh rằng $\frac{1}{a^2}+\frac{1}{b^2}+\frac{1}{c^2}+3\geq 2(a+b+c)$

 Áp dụng bất đẳng thức Schur bậc 3 và bất đẳng thức AM-GM ta có

$\begin{eqnarray*}\dfrac{1}{a^2}+\dfrac{1}{b^2}+\dfrac{1}{c^2}+3 &=& a^2b^2+b^2c^2+c^2a^2+3\sqrt[3]{a^4b^4c^4}\\ &\geq & \sum \sqrt[3]{a^2b^2.b^2c^2}(\sqrt[3]{a^2b^2}+\sqrt[3]{b^2c^2})\\ &=&\sum \sqrt[3]{b^4}(\sqrt[3]{a^2}+\sqrt[3]{c^2})\\ &\geq &2\sum \sqrt[3]{b^4ac}=2\sum a  \end{eqnarray*}$




#624271 $\sum \frac {ab}{1-ab}\leq \frac {3}{8}$

Đã gửi bởi Ankh on 02-04-2016 - 19:24 trong Bất đẳng thức - Cực trị

Cách dưới thì giống của mình rồi,còn cách làm Chebyshev thì tốt lắm :)

 Cách của mình :

 Ta có bất đẳng thức tương đương $\sum \left (\dfrac{4}{3}-\dfrac{1}{1-ab}\right )\geq \dfrac{5}{8}\Leftrightarrow \sum \dfrac{(1-4ab)^2}{(1-4ab)(1-ab)}\geq \dfrac{15}{8}$

 Do $1=(a+b+c)^2>(a+b)^2\geq 4ab$ nên áp dụng bất đẳng thức Cauchy-Schwarz ta có

 $\sum \dfrac{(1-4ab)^2}{(1-4ab)(1-ab)}\geq \dfrac{(3-4\sum ab)^2}{\sum (1-4ab)(1-ab)}=\dfrac{(3-4q)^2}{3-5q+4q^2-8r}\geq \dfrac{(3-4q)^2}{3-5q+4q^2-\dfrac{8(4q-1)}{9}}$

 Ta cần chứng minh $\dfrac{(3-4q)^2}{3-5q+4q^2-\dfrac{8(4q-1)}{9}}\geq \dfrac{15}{8}\Leftrightarrow (3q-1)(68q-41)\geq 0$

 Đúng do $q\leq \dfrac{1}{3}$

 -----------------

 Cái dạng này nhìn khá quen, không biết có lời giải bằng tiếp tuyến hoặc yếu tố không :)




#624293 $\sum \frac {ab}{1-ab}\leq \frac {3}{8}$

Đã gửi bởi Ankh on 02-04-2016 - 20:00 trong Bất đẳng thức - Cực trị

Mình không biết tại sao lại có ý tưởng thêm bớt $\frac{4}{3$ ở khúc này ? Bạn có thể giải thích không ?

 Mình đưa bất đẳng thức về chứng minh tương đương $\sum \left (k-\dfrac{1}{1-ab}\right )\geq 3k-\dfrac{27}{8}\Leftrightarrow \sum \dfrac{k-1-kab}{1-ab}\geq 3k-\dfrac{27}{8}$

 Dạng này nhìn có vẻ quen, vì các bài toán tương tự để giải bằng yếu tố hay Cauchy-Schwarz đếu có thể đưa về như vậy, và mình thêm bớt để có thể sử dụng Cauchy-Schwarz. Ta sẽ cần tìn một đánh giá cho $ab$ sao cho bất đẳng thức $k-1>kab$ càng chặt càng tốt. Ta có một số đánh giá như sau :

 $ab\leq \dfrac{(a+b)^2}{4}\leq \dfrac{a^2+b^2}{2}\leq \dfrac{1}{2}$

 $3(ab+bc+ca)\leq (a+b+c)^2$, cho $c\rightarrow 0$ thì thành $1\geq 3ab$

 $1=(a+b+c)^2>(a+b)^2\geq 4ab$

 Tuy nhiên nếu để ý một tí thì ta nhận ra rằng đánh giá $(a+b)^2\geq 4ab$ chặt hơn $a^2+b^2\geq 2ab$, và tuy rằng nó chưa phải chặt nhất ( vì theo mình biết thì còn có cả cái mà "Làm mạnh bất đẳng thức Cauchy" nữa ), lúc đó không biết do vô tình hay sao nhưng mình nghĩ $ab<\dfrac{1}{4}$ có lẽ là vừa đủ. 

 Tiếp theo và việc chọn hằng số $k$ sao cho $\dfrac{k-1}{1}=\dfrac{k}{4}\Leftrightarrow k=\dfrac{4}{3}$, việc tiếp theo như bạn thấy

 Nhưng thực ra mình nghĩ lời giải của mình ở trên vẫn mang hơi hướng gì đó hơi may mắn một tí, thực ra trong quá trình biến đổi mình có sai và tưởng như $\dfrac{1}{4}$ vẫn là không đủ. Và nói thật, lúc đó mình định buông xuôi :( May dò lại được :)




#624249 $\sum \frac {ab}{1-ab}\leq \frac {3}{8}$

Đã gửi bởi Ankh on 02-04-2016 - 18:15 trong Bất đẳng thức - Cực trị

Ai có lời giải thuần bằng C-S hay AM-GM thì post lên nhé :)

 

Xét phép đổi biến $p,q,r$,ta sẽ có $r\in \left ( 0;\frac{1}{27} \right ]; q \in \left ( 0;\frac{1}{3} \right ]$.Theo Schur thì $r\geqslant \frac{4q-1}{9}$

 

BĐT cần chứng minh tương đương với:

$\sum \frac{ab}{1-ab}\leqslant \frac{3}{8}\Leftrightarrow \sum \frac{1}{1-ab}\leqslant \frac{27}{8}\Leftrightarrow \frac{\sum (1-ab)(1-bc)}{(1-ab)(1-bc)(1-ca)}\leqslant \frac{27}{8}$

$\Leftrightarrow \frac{3-2q+r}{1-q+r-r^{2}}\leqslant \frac{27}{8}\Leftrightarrow f(r)=27r^{2}-19r+11q-3\leqslant 0$

 

Dễ thấy $f'(r)=54r-19<0,\forall r \in \left ( 0;\frac{1}{27} \right ]$ nên $f(r)\leqslant f\left ( \frac{4q-1}{9} \right )=\frac{(16q+5)(3q-1)}{9}\leqslant 0,\forall q \in \left ( 0;\frac{1}{3} \right ]$

 

Ta có đpcm.

 Có hai cách khác ở đây ạ http://diendantoanho...-xyleq-frac278/




#658509 $\sqrt{\frac{a^2}{a^2+b+c}}+...

Đã gửi bởi Ankh on 20-10-2016 - 09:54 trong Bất đẳng thức và cực trị

theo BĐT trên thì ta sẽ đi chứng minh : 

$\sum \frac{a^4}{1+a^2b}\geq \sum \frac{a^3b}{1+ab^2}\Leftrightarrow \sum \frac{a^3(a-b)}{(1+a^2b)(1+ab^2)}\geq 0$

Ta có : $\frac{a^3(a-b)}{X}+\frac{b^3(b-c)}{Y}+\frac{c^3(c-a)}{Z}=\frac{(a-b)^2(a^2+ab+b^2)}{M}+\frac{(b^3-c^3)(a-c)}{N}\geq 0$

BĐT trên đúng nếu giả sử $a\geq b\geq c$

BĐT đc CM $☺$

Có chắc là giả sử được không? :V




#658653 $\sqrt{\frac{a^2}{a^2+b+c}}+...

Đã gửi bởi Ankh on 21-10-2016 - 10:09 trong Bất đẳng thức và cực trị

Lâu rồi mới gặp lại mà nó làm khó t.

      Bài 1: Chứng minh rằng với mọi số thực $a,b,c$, ta có:

$2(1+abc)+\sqrt{2(1+a^2)(1+b^2)(1+c^2)} \geq (1+a)(1+b)(1+c)$

      Bài 2: Chứng minh rằng với mọi $a,b,c > 0$, ta có:

$\frac{a^4}{1+a^2b}+\frac{b^4}{1+b^2c}+\frac{c^4}{1+c^2a} \geq \frac{abc(a+b+c)}{1+abc}$

      Bài 3: Cho $a,b,c$ là các số thực dương thõa mãn $a^2+b^2+c^2=3$

                 Chứng minh rằng:

$\sqrt{\frac{a^2}{a^2+b+c}}+\sqrt{\frac{b^2}{b^2+c+a}}+\sqrt{\frac{c^2}{c^2+a+b}} \leq \sqrt{3}$

 Bài 2 làm như sau

 Áp dụng bất đẳng đẳng thức Cauchy-Schwarz:

 $\sum \dfrac{a^4}{1+a^2b}=\sum \dfrac{a^4bc}{bc+a^2b^2c}\geq \dfrac{abc\left(a\sqrt{a}+b\sqrt{b}+c\sqrt{c}\right)^2}{(1+abc)(ab+bc+ca)}$

 Nên ta chỉ cần chứng minh $\left(a\sqrt{a}+b\sqrt{b}+c\sqrt{c}\right)^2\geq (a+b+c)(ab+bc+ca)$

 Mà $ab+bc+ca\leq \dfrac{1}{3}(a+b+c)^2$

 Cho nên chỉ cần chỉ ra $3\left(a\sqrt{a}+b\sqrt{b}+c\sqrt{c}\right)^2\geq (a+b+c)^3$

 Lấy căn bậc 6 hai vế thì nó tương đương $\sqrt[3]{\dfrac{\sqrt{a}^3+\sqrt{b}^3+\sqrt{c}^3}{3}}\geq \sqrt{\dfrac{\sqrt{a}^2+\sqrt{b}^2+\sqrt{c}^2}{3}}$

 Đúng theo bất đẳng thức trung bình lũy thừa




#637390 4 - BĐT: Hàm số một biến

Đã gửi bởi Ankh on 01-06-2016 - 15:09 trong Bất đẳng thức - Cực trị

 

 Chuỗi 1:
 Cho x, y và z là ba số thực dương.
 Hãy tìm giá trị nhỏ nhất của biểu thức:
 
$P=\frac{1}{x+\sqrt{xy}+\sqrt[3]{xyz}}-\frac{2}{\sqrt{x+y+z}}.$

 

 Áp dụng bất đẳng thức AM-GM ta có $x+\sqrt{xy}+\sqrt[3]{xyz}\leq x+\dfrac{x+4y}{4}+\dfrac{x+4y+16z}{12}=\dfrac{4}{3}(x+y+z)$

 Đặt $\sqrt{x+y+z}=t$ thì $P\geq \dfrac{3}{4t^2}-\dfrac{2}{t}\geq \dfrac{-4}{3}$

 Dấu "=" xảy ra khi $16z=4y=x=\dfrac{3}{7}$

 

 

 

 Chuỗi 2:

 Cho x, y và z là ba số thực không âm, thoả mãn:

$xy+yz+zx=1.$
 Hãy tìm giá trị nhỏ nhất của biểu thức:
$P=\frac{1}{x^{2}+y^{2}}+\frac{1}{y^{2}+z^{2}}+\frac{1}{z^{2}+x^{2}}+\frac{5}{2}(x+1)(y+1)(z+1).$

 

 Giả sử $c=\min \{a,b,c\}$, khi đó ta có $\sum \dfrac{1}{x^2+y^2}\geq \dfrac{10}{(x+y+z)^2}$

 Lại có $(x+1)(y+1)(z+1)=xyz+1+x+y+z+xy+yz+zx\geq x+y+z+2$

 Đặt $t=x+y+z$ với $t\in [0;\sqrt{3}]$ thì $P\geq \dfrac{10}{t^2}+\dfrac{5(t+2)}{2}\geq \dfrac{35}{2}$

 Dấu "=" xảy ra khi $a=b=1,c=0$ cùng các hoán vị




#670196 $lim2^{n}\sqrt{2-u_{n}}$

Đã gửi bởi Ankh on 28-01-2017 - 02:12 trong Dãy số - Giới hạn

Cho dãy số $(u_{n})$ thoả mãn điều kiện: $u_{1}=\frac{\sqrt{6}-\sqrt{2}}{2},u_{n+1}=\sqrt{2+u_{n}}$ với mọi $n=1,2,...$. CMR: Dãy số $(u_{n})$ có giới hạn và tìm $lim2^{n}\sqrt{2-u_{n}}$

 Đặt $u_1=2\cos \dfrac{5\pi}{12}$ và sau đó quy nạp chứng minh được $u_n=2\cos \dfrac{5\pi}{3.2^{n+1}}$, rồi sử dụng giới hạn $\lim _{x\to 0} \dfrac{\sin x}{x}=1$ là xong




#632097 $d_{a}+d_{b}+d_{c}\leq \frac...

Đã gửi bởi Ankh on 09-05-2016 - 18:21 trong Hình học

:D

File gửi kèm  T8-463.doc   135K   218 Số lần tải